Q13

 
tzyc
Thanks Received: 0
Atticus Finch
Atticus Finch
 
Posts: 323
Joined: May 27th, 2012
 
 
trophy
Most Thankful
 

Q13

by tzyc Wed Apr 10, 2013 2:12 am

Why (A) cannot be the answer?
The author mainly talks about what both theories are and if they are combined, it allows to understand the socioeconomic mobility, right?
So "synthesis of approaches to an issue" sounds correct...
Or is it because s/he does not "advance" it? :|

Thank you
User avatar
 
noah
Thanks Received: 1192
Atticus Finch
Atticus Finch
 
Posts: 1541
Joined: February 11th, 2009
 
This post thanked 1 time.
 
 

Re: Q13

by noah Fri Apr 12, 2013 10:25 am

I had a moment of panic when I too found (A) compelling. It passed when I checked the answer key and saw it's the answer!

(A) summarizes the passage quite nicely - it's not just structural or just cultural issues, it's both!

(B) is unsupported since there's no tentative answer being challenged.

(C) is tempting perhaps, however there's no evaluation of theories, more a outlining of how they're both correct.

(D) is very close - it has two theories, people are happy at the end - however it doesn't have the "they're both right" that we need.

(E) is way too narrow. There's nothing about the theories.